help ! brainliest !!!

Help ! Brainliest !!!

Answers

Answer 1
answer:
a, the reflexive property

Related Questions

f(x) = |×| g(x) = |×+9| – 5 We can think of g as a translated (shifted) version of f. Complete the description of the transformation. Use nonnegative numbers. To get the function g. shift f up/down by units and to the right/left v by units.

Answers

First we can check the value 9 that is added inside the absolute module operator.

This value is together with the variable x inside the operator, that is, from f(x) we can replace x by 'x + 9' in order to get the first part of g(x)

This addition of 9 to the value of x represents a horizontal translation of 9 units to the left.

Then, the subtraction of 5 units outside the absolute module operator is a addition/subtraction to the function, that is, we can subtract 5 units from f(x) in order to get this second part of f(x).

This subtraction of 5 units to the value of f(x) represents a vertical translation of 5 units down.

So the answer is:

To get the function g(x), shift f down by 5 units and to the left by 9 units.​

Lana owns an office supply shop. At the beginning of each school year, she chooses two or three products to donate to the local middle school.
The table shows the school supplies that Lana has in her shop and how many of each kind she has in stock. Lana is considering different options of supplies to donate. For each option, determine the greatest number of identical boxes she could pack and the number of each supply item she could put in the boxes.

school supplies stock:
pencils-78
markers-110
notebooks-195
erasers-143
folders-330

A option 1: pencils and erasers: ____boxes with ____ pencils and ____ erasers

B option 2: notebooks and folders: ____ boxes with ____notebooks and ____ folders in each box.

C option 3: erasers, markers, and folders: ____ boxes with ____ erasers ____ markers and ____ folders in each box.

Answers

Answer: Step-by-step explanation: Lana needs to order supplies for the upcoming month. Her supplier sells pencils in boxes of 12, markers in boxes of 10, notebooks in boxes of 4, erasers in boxes of 6, and folders in boxes of 15. What is the least number of boxes of each supply item that Lana could order to have the same number of each item delivered? How many of each item will she get? E 3-2

f(x)=(0.13x⁴+0.22x³)-0.88x²-0.25x-0.09for this polynomial use a graph and find the minimum and maximum values written as coordinates

Answers

The given function is:

[tex]f(x)=0.13x^4+0.22x^3_{}-0.88x^2-0.25x-0.09[/tex]

The graph for the polynomial f(x) is shown below:

Let and y be whole-number variables such that y is the greatest whole number less than or equal to the table below lists some valuesfor and y.xy79411 513 6Which of the following statements is true?A. y changes by a constant amount when r changes by 2.OB. z changes by a constant amount when y changes by 1.C. y changes by a constant amount when changes by 1.D. 2 changes by a constant amount when y changes by 2.

Answers

we have

Verify each statement

A -------> For (7,3) and (9,4) -----> x changes by 2 and y change by 1

(9,4) and (11,5) -------> x changes by 2 and y change by 1

(11,5) and (13,6) -----> x changes by 2 and y change by 1

option A is true

B ----> option B is true

C -----> is not true

D ----> (7,3) and (11,5) ------> y changes by 2 and x changes by 4

(9,4) and (13,6) ----> y changes by 2 and x changes by 4

option D is true

step 2

Verify A, B and D

For x=14 ------> y=7

For x=16 -------> y=8

For x=18 ------> y=9

the answer must be option A

Find the product of these complex numbers.(8 + 6i)(-5 + 7i) =A.-82 - 86iB.-82 + 26iC.2 + 26iD.2 - 86i

Answers

Solution

Step 1:

Write the expression:

(8 + 6i)(-5 + 7i)

Step 2:

[tex]\begin{gathered} \left(8+6i\right)\left(-5+7i\right) \\ \\ 8\times(-5)\text{ + 8 }\times7i\text{ + 6i }\times(-5)\text{ + 6i }\times\text{ 7i} \\ \\ =\text{ -40 + 56i - 30i + 42i}^2 \\ \\ =\text{ -40 + 26i - 42} \\ \\ =\text{ - 82 + 26i} \end{gathered}[/tex]

Final answer

B.

-82 + 26i

What is the sum of the rational expressions below?3xХ+X+9 X-4O A.4x2-3xx2 +5X-36O B. 4x2-3x2x+5C.4x2x+5hosD.4xx2 +5x - 36

Answers

Given

[tex]\frac{3x}{x+9}+\frac{x}{x-4}[/tex]

To find the sum of the rational expressions.

Explanation:

It is given that,

[tex]\frac{3x}{x+9}+\frac{x}{x-4}[/tex]

Then,

[tex]\begin{gathered} \frac{3x}{x+9}+\frac{x}{x-4}=\frac{3x(x-4)+x(x+9)}{(x+9)(x-4)} \\ =\frac{3x^2-12x+x^2+9x}{x^2+9x-4x-36} \\ =\frac{4x^2-3x}{x^2+5x-36} \end{gathered}[/tex]

Hence, the answer is option A).

What is the lateral surface area of the of the following figure? A. 982.36B. 851.76C. 785.34D. 709.8

Answers

Given : a triangular prism

The lateral surface area is the sum of the rectangular sides

So, the lateral surface area =

[tex]\begin{gathered} 18.2\cdot13+18.2\cdot13+18.2\cdot20.8 \\ =851.76\operatorname{cm} \end{gathered}[/tex]

Another method : Find the perimeter of the triangle then multiply by 18.2

[tex]\begin{gathered} 18.2\cdot(13+13+20.8)_{} \\ =18.2\cdot46.8 \\ =851.76\operatorname{cm} \end{gathered}[/tex]

so, the answer is option B. 851.76

I need to know if A and B are CORRECT and I need HELP WITH C

Answers

The sum of all angles of a triangle is always equal to 180°. Therefore, looking at triangle A we have 3 angles, 53°, 68°, and x. The sum of them will be 180:

a)

[tex]x+53+68=180[/tex]

Now we solve it for x

[tex]\begin{gathered} x=180-53-68 \\ \\ x=59\degree \end{gathered}[/tex]

b)

Same thing here, the angles are 46°, 71°, and x, the sum is equal to 180°:

[tex]\begin{gathered} x+46+71=180 \\ \\ x=180-46-71 \\ \\ x=63\degree \end{gathered}[/tex]

c)

Now for C, we have the angles x, x again, and 24°, therefore the sum will be

[tex]\begin{gathered} x+x+24=180 \\ \\ 2x+24=180 \\ \\ 2x=180-24 \\ \\ 2x=156 \\ \\ x=\frac{156}{2} \\ \\ x=78\degree \end{gathered}[/tex]

Final answers:

a) x = 59°

b) x = 63°

c) x = 78°

2. The following triangle is an isosceles triangle. What is the length of the missing side? ? 11 in. 37 ? 4 in. 11 in 37° 4 in 530

Answers

An ISOSCELES triangle has two sides equal, and two base angles are also equal.

The two sides on the left and the right are equal. The right side measures 11 inches, therefore the left side also measures 11 inches.

The correct answer option is 11 inches

9. A gallon of lemonade calls for 2 scoops of sugar. If you want to make 5 gallons, how much sugar should you put in? (2 pts)​

Answers

Answer:10

Step-by-step explanation: if you need 2 for 1 gallon multiply it by 5

you have 1 gallon with 2 scoops

2 for 5 gallons gives you 10 scoops total for 5 gallons

Rearrange the equation so r is the independent variable q-10=6(r+1)

Answers

[tex]\begin{gathered} The\text{ equation is,} \\ q-10=6(r+1) \\ q-10=6r+6 \\ 6r=q-10-6 \\ 6r=q-16 \\ r=\frac{(q-16)}{6} \end{gathered}[/tex]

2 A cognitive psychologist conducted a study of whether familiarity of words (X) predicts the time it takes (in seconds) to press a button indicating whether the word is singular or plural (I), with all participants being given the same words. Familiarity with these words was rated at a later time on a 7-point scale (with higher numbers indicating more farniliarity). The participants' scores were 6 2 5 3 7 Y 0.3 1.5 0.8 1.4 0.1 a Figure the Pearson correlation coefficient (25 pts.).

Answers

We will have the following:

*Firts: We have that the correlation coefficient is given by:

[tex]r=\frac{\sum ^n_{i=1}X_iY_i-\frac{1}{n}(\sum ^n_{i=1}X_i)(\sum ^n_{i=1}Y_i)}{\sqrt[]{\sum^n_{i=1}X^2_i-\frac{1}{n}(\sum^n_{i=1}X_i)^2}\sqrt[]{\sum ^n_{i=1}Y^2_i-\frac{1}{n}(\sum ^n_{i=1}Y_i)^2}}[/tex]

*Second: We calculate the means, that is:

[tex]x_m=4.6[/tex]

&

[tex]y_m=\text{0}.82[/tex]

*Third: We calculate the sums:

[tex]\sum ^n_{i=1}X^2_i-\frac{1}{n}(\sum ^n_{i=1}X_i)^2=123-\frac{23^2}{5}=17.2[/tex][tex]\sum ^n_{i=1}Y^2_i-\frac{1}{n}(\sum ^n_{i=1}Y_i)^2=4.95-\frac{4.1^2}{5}=1.588[/tex][tex]\sum ^n_{i=1}X_iY_i-\frac{1}{n}(\sum ^n_{i=1}X_i)(\sum ^n_{i=1}Y_i)=13.7-\frac{23\cdot4.1}{5}=-5.16[/tex]

Fourth: We replace the data:

[tex]r=\frac{-5.16}{\sqrt[]{17.2\cdot1.588}}\Rightarrow r=-0.987[/tex]

Thus making the coefficient r = -0.987.

And this would be the scatterplot:

Given the matrices A and B shown below, find – B – 1/3A[ -18 3]. [ -4 12][ -15 -6] [ 8 -12]

Answers

Answer:

[10 -13]

[-3 14]

Explanation:

First, we will calculate 1/3A, so:

[tex]\frac{1}{3}A=\frac{1}{3}\begin{bmatrix}{-18} & 3 & \\ {-15} & -6 & {} \\ {} & {} & {}\end{bmatrix}=\begin{bmatrix}{\frac{1}{3}(-18)} & {\frac{1}{3}(3)} & \\ {\frac{1}{3}(-15)} & {\frac{1}{3}(-6)} & {} \\ {} & {} & {}\end{bmatrix}=\begin{bmatrix}{-6} & {1} \\ {-5} & {-2} \\ & {}\end{bmatrix}[/tex]

Because 1/3 multiply each value in the matrix. Now, adding the respective values in the same position, we can calculate -B - 1/3A as:

[tex]\begin{gathered} -B-\frac{1}{3}A=-\begin{bmatrix}{-4} & {12} & \\ {8} & {-12} & {}{}\end{bmatrix}-\begin{bmatrix}{-6} & {1} \\ {-5} & {-2}\end{bmatrix} \\ -B-\frac{1}{3}A=\begin{bmatrix}{4} & {-12} & \\ {-8} & {12} & {}{}\end{bmatrix}-\begin{bmatrix}{-6} & {1} \\ {-5} & {-2}\end{bmatrix} \\ -B-\frac{1}{3}A=\begin{bmatrix}{4-(-6)} & {-12-1} & \\ {-8-(-5)} & {12-(-2)} & {}{}\end{bmatrix} \\ -B-\frac{1}{3}A=\begin{bmatrix}{4+6} & {-12-1} & \\ {-8+5} & {12+2} & {}{}\end{bmatrix} \\ -B-\frac{1}{3}A=\begin{bmatrix}{10} & {-13} & \\ {-3} & {14} & {}{}\end{bmatrix} \end{gathered}[/tex]

Therefore, the answer is:

[tex]-B-\frac{1}{3}A=\begin{bmatrix}{10} & {-13} & \\ {-3} & {14} & {}\end{bmatrix}[/tex]

what is the domian of the graph

Answers

answer:

2 ≤ x ≤ 5

step-by-step explanation:

at the end of the piece-wise function (the short line), the circles are closed.

closed circles = ≤ or ≥

open circles = < or >

immediately, c and d are out of the equation.

the domain refers to what the range of x-values is (bottom line) so let's look. we have (2,5) and (5,3)

the x-values of those are 2 and 5, with x in between/including them.

(if we were to find the range it would be 5 and 3, with x in between/including them)

so, 2 ≤ x ≤ 5. we use ≤ because it is not smaller than 2, and not bigger than five. using ≥ would be all numbers except in between 2 and 5.

$5,500 how much money would be in the savingaccount after 5 years if the compounds interest monthly at a rate of 5% per year.

Answers

Answer:

There would be $7,058.47 in the saving account.

Step-by-step explanation:

The amount of money, after t years, with compound interest, is given by the following formula:

[tex]A(t)=P(1+\frac{r}{n})^{n\ast t}[/tex]

In which:

P is the amount of the initial deposit.

r is the interest rate, as a decimal.

n is the number of compoundings per year.

t is the number of years.

In this question:

Deposit of $5,500, so P = 5500.

5 years, so t = 5.

Rate of 5%, so r = 0.05.

Monthly compounding, so 12 times a year, which means that n = 12.

Then

[tex]A(5)=5500(1+\frac{0.05}{12})^{12\ast5}=7058.47[/tex]

There would be $7,058.47 in the saving account.

Find the equation of a line passing through the points (3,-4) and (1,2)

Answers

Explanation:

The equation of a line in slope-intercept form is:

[tex]y=mx+b[/tex]

Where m is the slope and b is the y-intercept.

The formula for the slope of a line with points (x1, y1) and (x2, y2) is:

[tex]m=\frac{y_1-y_2}{x_1-x_2}[/tex]

In this case the slope is:

[tex]m=\frac{-4-2}{3-1}=\frac{-6}{2}=-3[/tex]

For now we have:

[tex]y=-3x+b[/tex]

To find the y-intercept b, we have to replace (x,y) for one of the given points and solve for b. If we use point (1,2):

[tex]\begin{gathered} y=-3x+b \\ \text{ replacing x = 1 and y = 2} \\ 2=-3\cdot1+b \\ 2=-3+b \\ b=2+3=5 \end{gathered}[/tex]

Answer:

The equation of the line is: y = -3x + 5

Answer:

y=\neg;[3]x+5

Step-by-step explanation:

At a sand and gravel plant, sand is falling off a conveyor and onto a conical pile at a rate of 4 cubic feet per minute. The diameter of the base of the cone is approximately three times the altitude. At what rate is the height of the pile changing when the pile is 22 feet high?

Answers

[tex]h^{\prime}=\frac{4}{1089\pi}ft\text{ /min}[/tex]

STEP - BY - STEP EXPLANATION

What to find?

dh/dt

Given that;

At a sand and gravel plant, sand is falling off a conveyor and onto a conical pile at a rate of 4 cubic feet per minute. The diameter of the base of the cone is approximately three times the altitude.

Since we have that the rate is 4 cubic per minute, then dv/dt = 4 (since v is the volume of the cone at time t.)

The formula for the volume of a cone is

[tex]V=\frac{1}{3}\pi r^2h---------------(1)[/tex]

Where r is the radius and h is the height.

We have that, the diameter of the cone is approximately three times the altitude.

That is;

diameter = 3h

But, d = 2r

⇒2r = 3h

r = 3h/2

Now, we substitute r=3h/2 into equation (1).

[tex]V=\frac{1}{3}\pi(\frac{3h}{2})^2h[/tex][tex]=\frac{1}{3}\pi(\frac{9h^2}{4})h[/tex][tex]V=\frac{3}{4}\pi h^3[/tex]

Now, differentiate the above with respect to t.

[tex]\frac{dv}{dt}=\frac{3}{4}\pi3h^2\frac{dh}{dt}[/tex]

Simplify .

[tex]\frac{dv}{dt}=\frac{9}{4}\pi h^2\frac{dh}{dt}[/tex]

Make dh/dt subject of formula.

[tex]\frac{dh}{dt}=\frac{dv}{dt}\times\frac{4}{9\pi h^2}----------(2)[/tex]

Recall that, dv/dt = 4 and h=22

Substituting the values into equation (2), we have;

[tex]\frac{dh}{dt}=4\times\frac{4}{9\pi(22)^2}[/tex][tex]=\frac{16}{9\pi\text{ (484)}}[/tex][tex]=\frac{4}{9\pi\text{ (121)}}[/tex][tex]=\frac{4}{1089\pi}[/tex]

Therefore, h' = dh/dt = 4/1089π ft/min.

Algebra 1 Question. Please View Attachment. Help needed ASAP :)

Answers

Recall that the graph of h(x) translated n units to the right is the graph of h(x-n).

Now, notice that the given graph is the graph of the function f(x)=x translated 1 unit to the right, therefore the given graph is the graph of f(x-1).

Setting f(x+k)=f(x-1), since the graph is a line we get that:

[tex]x+k=x-1.[/tex]

Subtracting x from the above equation we get:

[tex]k=-1.[/tex]

Answer: First option.

Show me how do divided 6.345 ÷ 0.09 step by step.

Answers

70.5

1) To divide a decimal number by another one, we can turn them into fractions and simplify it, whenever possible:

[tex]\begin{gathered} 6.345=\frac{6345}{1000} \\ 0.09=\frac{9}{100} \end{gathered}[/tex]

Each decimal place represents the number of zeros beside the 1 on the denominator, or each decimal place represents 1/10.

2) Dividing it as fractions, we need to multiply the first by its reciprocal:

[tex]\frac{\frac{6345}{1000}}{\frac{9}{100}}=\frac{6345}{1000}\times\frac{100}{9}=\frac{6345}{90}=\frac{141}{2}=70.5[/tex]

Notice that we simplified 6345/90 by 45 then we got 141/2. In other words, 6345/100 and 141/2 are the same, Because 141/2 is the nonreducible fraction.

To divide 6.345 ÷ 0.09 using long division, let's multiply first both numbers by 100, doing this we'll keep the proportionality, and make our calculations easier.

6.345 x 100 = 634.5

0.09 x 100 = 9

1) First, let's divide 634 by 9. 70 x 9 = 630. The closest value.

2) Then let's bring down 4.5. 4.5 ÷ 9 = 0.5

3) 70.5 x 9 = 634.5.

Kris and Pat were born on the exact same day but not in the same year their ages are shown in the table When Pat was 30 how old was Chris

Answers

A ) 18 years old

B) 19 years old

C) 27 years old

add three to kris' age to find Pat's age

Explanation

Step 1

when Kris was 15 .how old was pat

find the rule

a) check if the difference between the ages is constant

[tex]\begin{gathered} \text{Pat's age - Kris' Age=} \\ 7-4=3 \\ 10-7=3 \\ 15-12=3 \\ y_4\text{-}15= \\ 22-x_5= \\ 26-23=3 \end{gathered}[/tex]

it means Pat is 3 years older than Kriss

then

when Kris was .how old was pat

[tex]\begin{gathered} y_4\text{-}15=3 \\ y_4-15=3 \\ \text{add 15 in both sides} \\ y_4-15+15=3+15 \\ y_4=18 \end{gathered}[/tex]

Step 2

when Pat was 22, How old was Kris

the difference between the ages is the same, 3

so

[tex]\begin{gathered} 22-x_1=3 \\ \text{subtract 22 in both sides} \\ 22-x_5-22=3-22 \\ -x_5=-19 \\ x_5=19 \end{gathered}[/tex]

Step 3

when Pat was 30, How old was kris

replace

[tex]\begin{gathered} \text{Pat's age -Kris' age=3} \\ 30-\text{Kri's age=3} \\ \text{subtract 30 in both sides} \\ 30-\text{Kri's age-30=3-}30 \\ -\text{Kris'age =-27} \\ \text{Kris'age }=27 \end{gathered}[/tex]

Step 4

wich choice best represents the rule

as we saw before,

add three to kris' age to find Pat's age

[tex]\text{Kris' age+3}=\text{Pat's age}[/tex]

Problems 20 - 23. Analytically determine what type(s) of symmetry, if any, the graph of the equation would possess. Show your work.21) y^2 - xy = 6

Answers

Because of gthe graph I conclude that the graph is simmetric about the origin.

Polygon ABCD with A (2,4), B (-4,-8), C (0,4), and D (12,-2), is dilated are the new coordinates? Is this a reduction or enlargement?

Answers

For this exercise it is important to know that, in Dilations:

1. When the scale factor is greater than 1, the Image (the figure obtained after the transformation) is an enlargement.

2. When the scale factor is greater than 0 but less than 1, the Image is a reduction:

[tex]0You know that the coordinates of the vertices of the polygon ABCD are:[tex]\begin{gathered} A(2,4) \\ B(-4,-8) \\ C(0,4) \\ D(12,-2) \end{gathered}[/tex]

And the scale factor is:

[tex]k=\frac{1}{2}[/tex]

Since:

[tex]0<\frac{1}{2}<1[/tex]

It is a reduction.

You can identify that the rule of this transformation is:

[tex]undefined[/tex]

9. In April, Community Hospital reported 923 discharge days for adults and children and 107 discharge daysfor newborns. During the month, 192 adults and children and 37 newborns were discharged. Calculate theALOS for adults and children for the month of April. Round to one decimal place.

Answers

Solution:

Average Length of Stay(ALOS) is calculated by summing the number of days for all stays (where partial days, including non-overnight stays, are rounded up to the next full day) and dividing by the number of patients.

[tex]\text{alos}=\frac{total\text{ number of days}}{\text{totl number of patients}}[/tex]

The total number of days is

[tex]\begin{gathered} =923+107 \\ =1030 \end{gathered}[/tex]

The total number of patients is

[tex]\begin{gathered} =192+37 \\ =229 \end{gathered}[/tex]

By applying the formula above, we will have

[tex]\begin{gathered} \text{alos}=\frac{total\text{ number of days}}{\text{total number of patients}} \\ \text{alos}=\frac{1030}{229} \\ \text{alos}=4.4978 \\ \text{alos}\approx4.5 \end{gathered}[/tex]

Hence,

The final answer is =4.5

Use the following information to fill out the entire two-way table.At PRHS, there are 450 students in the 9th and 10th grade taking geometry, and one third ofthem are 9th graders. The students were surveyed on which unit from quarter 4 they liked best.65 students said that unit 5 was their favorite, but only 25 of them were 9th graders. Unit 8 wasthe most popular for 9th graders, with 50 of them saying it was their favorite. Unit 7 was themost popular with 10th graders, with 100 of them saying it was their favorite. Unit 6 and Unit 8were equally popular for 10th grade students. A total of 125 students sald that Unit 6 was theirfavorite.Answer ALL 3 of the following questions.1. What is the probability that a randomly selected student will be a 9th grade student OR astudent that preferred unit 7? Show your work or explain how you know. Leave it insimplified fraction form.2. What is the probability that a randomly selected student will be a 10th grade student whoalso prefers unit 8? Show your work or explain how you know. Leave it in simplifiedfraction form.3. Given the student prefers Unit 5, what is the probability the student is in the 10th grade?Show your work and explain how you know. Leave it in simplified fraction form.

Answers

[tex]\begin{gathered} 1)\text{ }\frac{28}{45} \\ \\ 2)\text{ }\frac{8}{45} \\ \\ 3)\text{ }\frac{40}{65} \end{gathered}[/tex]

Here, we want to calculate probabilities;

We have this as follows;

1) We want to calculate the probability that a randomly selected student is a 9th grader or a student that preferred unit 7

From here, we need the number of students who are 9th graders and students that prefer unit 7

From the question, we have it that 1/3 of the total students are 9th graders

So, for a total of 450, the number of 9th graders will be 1/3 * 450 = 150 students

Secondly we need the number of students that prefers unit 7

Let us try and complete the table as follows;

From the completed table, the numbers that like unit 7 are 130

So the probability we want to calculate is the sum of the two divided by 450

We have this as;

[tex]\frac{130+150}{450}\text{ = }\frac{280}{450}\text{ = }\frac{28}{45}[/tex]

2) Here, we want to calculate the probability that a randomly selected student is a 10th grader who also prefers unit 8

From the table, we can see that the number of students who are 10th graders and also prefer unit 8 is 80

So, we have the probability as;

[tex]\frac{80}{450}\text{ = }\frac{8}{45}[/tex]

3) Here, we want to calculate the probability that given that a student prefers unit 5, what is the probability that he is a 10th grader

We use the conditional probability value here

Where event A is the probability that student is a 10th grader, while event B is the probability that a student prefers unit 5

We have the probability as;

[tex]\begin{gathered} P(A|B)\text{ = }\frac{P(AnB)}{P(B)} \\ \\ P(\text{AnB) = }\frac{40}{450};\text{ P(B) = }\frac{65}{450} \\ \\ P(A|B)\text{ = }\frac{40}{65} \end{gathered}[/tex]

Calculate the simple interest due on a 54-day loan of $3600 if the interest rate is 3%. (Round your answer to the nearest cent.)

Answers

Answer:

$5832.00cents

Step-by-step explanation:

PRT÷100

$3600 ×54 ×3 /100

= $5832.00cents.

The value of a ratio is 4/3. The second quantity in the ratio is how many times the first quantity in the ratio?

Answers

The second quantity in the ratio is 3/4 times the first quantity in the ratio.

How many times in the second quantity in the ratio more than the first quantity?

Ratio is the number of times that one value is contained within other value(s). This ratio is expressed as an improper fraction. A fraction is a non-integer that is made up of a numerator and a denominator. An improper fraction is a fraction in which the numerator is larger than the denominator.

In order to determine the number of times the second quantity is greater than the first quantity, determine the inverse of the given fraction.

The inverse of 4 /3 is 3/4.

To learn more about ratios, please check: https://brainly.com/question/25927869

#SPJ1

Solve the equation using the Complete The Square Methodx^2+12x=13Do your work on paper. Take a picture, and upload it here. Show all your work/steps.

Answers

Given the quadratic equation:

[tex]x^2+12x=13[/tex]

We can rewrite the equation as follows:

[tex]\begin{gathered} x^2+12x-13=0 \\ x^2+2\cdot6\cdot x-13=0 \\ x^2+2\cdot6\cdot x+6^2-6^2-13=0 \\ (x^2+2\cdot6\cdot x+6^2)-36-13=0 \end{gathered}[/tex]

We see that the term inside the parenthesis is a perfect square polynomial. Then:

[tex](x+6)^2-49=0[/tex]

Solving for x:

[tex](x+6)^2=49[/tex]

Taking the square on both sides:

[tex]\begin{gathered} \sqrt{(x+6)^2}=\sqrt{49} \\ \\ |x+6|=7 \end{gathered}[/tex]

This equation can turn into two equations:

[tex]\begin{gathered} x+6=7...(1) \\ x+6=-7...(2) \end{gathered}[/tex]

Solving (1):

[tex]\begin{gathered} x=7-6 \\ \\ \Rightarrow x=1 \end{gathered}[/tex]

Solving (2):

[tex]\begin{gathered} x=-7-6 \\ \\ \Rightarrow x=-13 \end{gathered}[/tex]

Finally, the solutions to the equation are:

[tex]\begin{gathered} x_1=1 \\ \\ x_2=-13 \end{gathered}[/tex]

According to psychologists, IQs are normally distributed, with a mean of 100 and a standard deviation of 18.a. What percentage of the population has IQs between 82 and 100?

Answers

To solve this question, we will have to find the Z score.

[tex]Z=\frac{x-\bar{x}}{s.d}[/tex]

Where x is the value of the IQ

X-bar is the mean.

s.d is the standard deviation

[tex]\begin{gathered} Z_1=\frac{82-100}{18} \\ =-\frac{18}{18} \\ =-1 \end{gathered}[/tex][tex]Z_2=\frac{100-100}{18}[/tex][tex]\begin{gathered} Z_2=\frac{0}{18} \\ =0 \end{gathered}[/tex]

The percentage of the population with IQs between 82 and 100 will be calculated thus:

[tex]\begin{gathered} P(-1-1) \\ =0.5-0.1587 \\ =0.34134 \\ \text{The percentage is}\colon \\ =0.34134\times100=34.134\text{ \%} \end{gathered}[/tex]

Given right triangle ABC with altitude BD drawn to hypotenuse AC. If AC = 15 and DC = 6, what is the length of BC in simplest radical form? (Note: the figure is not drawn to scale.) B х A D 6 C с 15 Submit Answer Answer: I

Answers

Answer

BC = x = 3√10

Explanation

To answer this question, we will use the concept of similar triangles.

We know that the two triangles ABC and BDC are similar because they are right angle triangles with one common non-right angle angle too, Angle C.

Using angle C as a reference point, we can write the corresponding sides.

And we know that corresponding sides for similar triangles have the same ratio.

∆ABC = ∆BDC

AB is corresponding to BD

BC is corresponding to DC

CA is corresponding to CB

So,

(AB/BD) = (BC/DC) = (CA/CB)

The sides that we need include

BC, DC, CA and CB

BC = x

DC = 6

CA = 15

CB = x

(BC/DC) = (CA/CB)

(x/6) = (15/x)

Cross multiply

x² = (6)(15)

x² = 90

Take the square root of both sides

√(x²) = √(90)

x = √90

x = √[(9)(10)]

x = (√9) (√10)

x = 3√10

Hope this Helps!!!

The probability is
(Round to four decimal places as needed.)
Points: 0 of 1
Save
Assume that when human resource managers are randomly selected, 43% say job applicants should follow up within
two weeks. If 7 human resource managers are randomly selected, find the probability that at least 2 of them say job
applicants should follow up within two weels.

Answers

Using the binomial distribution, it is found that there is a 0.7873= 78.73% probability that at least 2 of them say job applicants should follow up within two weeks.

For each manager, there are only two possible outcomes. Either they say job applicants should follow up within two weeks, or they do not say it. The opinion of a manager is independent of any other manager, which means that the binomial distribution is used to solve this question.

Binomial probability distribution  

P(X = x) = Cₙ,ₓ.pˣ.(1-p)ⁿ⁻ˣ

Cₙ,ₓ = n!/x!

The parameters are:

n is the number of trials.

x is the number of successes.

p is the probability of a success on a single trial.

In this problem:

43% say job applicants should follow up within two weeks. If 7 human resource managers are randomly selected, n = 8

The probability that at least 2 of them say job applicants should follow up within two weeks is P(X≥2), which is given by,

P(X≥2) = 1 - P(X < 2)

In which:

P(X<2)=P(X=0)+P(X=1)

Then,

P(X=x) = Cₙ,ₓ.pˣ.(1-p)ⁿ⁻ˣ

P(X=0) = C₇,₀.(0.43)°.(0.57)⁷ = 0.0195

P(X=1) = C₇,₁.(0.43)¹.(0.57)⁶ = 0.1032

P(X<2) = P(X=0)+P(X=1)

= 0.0195+0.1032

= 0.2127

P(X≥2) = 1-P(X<2)

= 1 - 0.2127

= 0.7873

0.7873 = 78.73% probability that at least 2 of them say job applicants should follow up within two weeks.

Hence we get the probability as 78.73%.

Learn more about Permutations and Combinations here:

brainly.com/question/25166697

#SPJ9

Other Questions
Choose the response that correctly replaces the underlined portion of the following phrase:Nous sommes dans une station de mtro.Vous tes dans lesquelles?Vous tes dans laquelle ?Vous tes dans lequel?Vous tes dans lesquels ? #32 At 10am, a green car leaves a house at a rate of 60 mph. At the same time, a blue carleaves the same house at a rate of 50 mph in the opposite direction. At what time will the carsbe 330 miles apart?#3b. Two bicyclists ride in the same direction. The first bicyclist rides at a speed of 8 mph.One hour later, the second bicyclist leaves and rides at a speed of 12 mph. How long will thesecond bicyclist have traveled when they catch up to the first bicyclist? simplifyx^-1 X (y^-8 X z^5)^3------------------------------x^-4 X y^-3 X z^6 Suppose you found a CD that pays 2.1% interest compounded monthly for 6 years. If you deposit $12,000 now, how much will you have in the account in 6 years? (Rounded to the nearest cent.) What was the interest earned? Now suppose that you would like to have $20,000 in the account in 6 years. How much would you need to deposit now? 1. Determine the domain and range of the quadratic function shown in the graph and represent the domainand range using inequalities. Solve the following system of equations by graphing. y = 12x 2 y = 32x + 2 Find the next term of the geometric sequence 3/2, 3/4, 3/8 , ... The map above is a road map of Learner County. Each centimeter on the map represents 30 miles. About how far is it from the town of Presley to Mt. Sametone on the road shown in red?A. 270 milesB. 180 milesC. 360 milesD. 390 miles PLEASE HELP DUE SOON ONLY Q6 I have the work for the rest. Solve the systems using subsitution for 1 and 2Solve the system using elimination for question 3 A loan of $43,000 is made at 5.25% interest, compounded annually. After how many years will the amount due reach $64,000 or more? (Use the calculator provided if necessary.)Write the smallest possible whole number answer. Determine whether triangle DEF with vertices D(6, -6), E(39, -12), and F(24, 18) isscalene (no congruent sides), isosceles (two congruent sides), or equilateral (threecongruent sides). 4. There are several biomes that are not marked "diverse" or "non-diverse." How would you describe those biomes? (3 points) Write the percent as a decimal. Round to the nearest thousandth if the division does not terminate.5/40%= clarify each of the following triangles by their angles in sides How are fiber optic cables and traditional, metal-based wires similar and different? Find the GCF : (7x^2y ,x^2y^2,21x^4y^2) Were there any famous party leaders for the workers party (1921)? Question 5 of 12SubmitWhich one of the following is an example of the First Law of Thermodynamics?A) As time passes, your room becomesmessier unless you spend energy to clean it.B) Water will only become ice if it is in a verycold environment.C) When a combustion reaction producesheat, that heat is absorbed by the air aroundthe reaction.D) A chemical ice pack feels cold because it isreducing the energy of the universe. a particular substance is typically found in nature as a pure element. what can you conclude about the reactivity of this element? (hint: which family contains elements that are generally found as pure elements?)